LSAT and Law School Admissions Forum

Get expert LSAT preparation and law school admissions advice from PowerScore Test Preparation.

 OneSeventy2019
  • Posts: 18
  • Joined: Sep 09, 2019
|
#74250
Powerscore,

I struggled to identify E as the correct answer choice to this question. I was able to use POE to eliminate choices A,B,C and D as irrelevant, so I ultimately got this question right.

That being said, however, I was not sure how the facts presented in stimulus would lead one to object to the program proposed in choice E. The only thing that I could think of is that the argument claimed that "government-subsidized insurance available to homeowners makes it feasible for anyone...". This got me thinking, 'well, if anyone can be insured then there is no reason to create another program for the uninsured since, what's their excuse for not having insurance in the first place?'.

Am I on the right track here?

Thanks!
 Paul Marsh
PowerScore Staff
  • PowerScore Staff
  • Posts: 290
  • Joined: Oct 15, 2019
|
#74256
Hi OneSeventy2019! You are exactly right. The first four answer choices are irrelevant (no support for an argument against them is provided by the passage). On the other hand, it provides the basis for an argument against the proposal Answer Choice (E) similar to the one you began describing. The argument against (E) using the information contained in the stimulus would probably go something like, "We already subsidize homeowners insurance for these people and the insurance covers basically all losses, so there's no excuse for anyone to not have insurance. Plus, forming this contingency fund would create a disincentive for coastal owners to purchase homeowners insurance."

Although I think a candidate or two in the current Democratic primary might take umbrage at that line of reasoning! Anyways, you're right on the money - (E) is the only relevant answer. Hope that helps.
 apimlott
  • Posts: 4
  • Joined: Sep 09, 2022
|
#98953
I'm having trouble understanding this question in its entirety. For starters, I do not understand why this question is classified as a "cannot" question. I thought the B was the correct answers because I understood the argument to be working toward some argument about the effective use of funds. I can see why B would be weak, but I cannot understand why E is a strong answer.

Could someone provide a more full explanation of this question and each of the answer choices?

Thank you.
 Rachael Wilkenfeld
PowerScore Staff
  • PowerScore Staff
  • Posts: 1358
  • Joined: Dec 15, 2011
|
#98968
Sure, apimlott.

Our stimulus gives us a set of facts about buildings in hurricane zones. The government subsidizes insurance that helps people who otherwise couldn't shoulder the risk of loss from a hurricane. The storms cause billions of dollars worth of damage and the insurance makes it so that the homeowners with the insurance can recoup a high percentage of their costs.

This means that the government is subsidizing the risk of building in these areas because those without the means to shoulder the loss themselves are able to get subsidized insurance to help cover the risk. This system is inefficient--it means that those that take the risk (the homeowners) aren't the same ones paying for that risk (the government) .

The question stems asks which of the policies would be one that the stimulus would argue against. That's another way of thinking about what policy is inconsistent with the view on risk suggested in the stimulus. So we are looking for a situation where the government would be covering the risk for a specific group---where those taking the financial risk are not the same as those engaging in the risky activity.

Answer choice (A): This is an example of a situation meant to reduce risky behavior. Government funding to reduce the potential risk of harm/loss is not what we are looking for.

Answer choice (B): This is another government program to reduce risk---warning systems don't increase people taking risky behaviors. They help reduce to risk of loss.

Answer choice (C): Emergency life support is also not something that would motivate people to engage in risky behavior. Rarely do people willingly engage in behavior they know will result in requiring emergency life support. Therefore the policy wouldn't result in an increase in risky behavior.

Answer choice (D): This is something that the facts don't really have any bearing on. The ecologically responsible is different than financial risk or property damage.

Answer choice (E): This is the correct answer choice. This would be a policy where the government is subsidizing risk. Those who don't even take the basic precaution of purchasing insurance are still having their riskier behavior subsidized by the government. The statements in the stimulus suggest why it's a bad idea for the government to subsidize risky behavior. The author of the stimulus would argue strongly against this policy, so it's our correct answer choice.

Hope that helps!

Get the most out of your LSAT Prep Plus subscription.

Analyze and track your performance with our Testing and Analytics Package.